Gruppo generato?

Analisi, algebra lineare, topologia, gruppi, anelli, campi, ...
Rispondi
sqrt2
Messaggi: 142
Iscritto il: 19 gen 2006, 14:43
Località: Genova

Gruppo generato?

Messaggio da sqrt2 »

Ho cambiato il primo vecchio post per presentare il problema nella sua formulazione estesa:

$ U_n := (U(\mathbb{Z}_n),*) $ è ciclico se e solo se $ n = 2, 4, p^m, 2p^m $ con $ p $ primo dispari.

Una dimostrazione quasi completa è nell'ultimo post.

(Vecchia formulazione: Dimostrare che $ \mathbb{Z}_p^* $ rispetto al prodotto è un gruppo ciclico.)
Ultima modifica di sqrt2 il 13 ott 2008, 23:01, modificato 2 volte in totale.
sqrt2
Messaggi: 142
Iscritto il: 19 gen 2006, 14:43
Località: Genova

Messaggio da sqrt2 »

(Double Hint:
- considerare $ H=\{x \in \mathbb{Z_p ^*}\ : ord(x)~\mid ord(a) $con $ a $ elemento di ordine massimo di $ \mathbb{Z_p ^*} $
- sfruttare il fatto (da dimostrare) che in un gruppo commutativo, se esistono due elementi di ordine m e n, allora ce n'è uno di ordine mcm(m,n))
Ultima modifica di sqrt2 il 13 ott 2008, 22:29, modificato 1 volta in totale.
Avatar utente
moebius
Messaggi: 433
Iscritto il: 08 mag 2005, 19:14

Messaggio da moebius »

Penso che il problema non sia la difficoltà, ma il fatto che la dimostrazione è abbastanza nota... Almeno (credo) per chi ha fatto un corso di algebra.
Tanto per fare un esempio, la dimostrazione che conosco io è quella che "suggerisci" tu.
Visto che in questa sezione in teoria bazzicano più universitari che liceali e che gli ingegneri di solito sono poco interessati dall'algebra...
Però magari mi sbaglio.
Se però avessi ragione sarebbe carino trovare dimostrazioni alternative... Dopo la dimostrazione topologica dell'infinità dei primi non mi stupisco più di niente :D
Fondatore: [url=http://olimpiadi.dm.unipi.it/oliForum/viewtopic.php?t=8899]Associazione non dimenticatevi dei nanetti![/url]
Membro: Club Nostalgici
Sono troppo scarso in italiano per usare parole con la c o la q...
Avatar utente
Nonno Bassotto
Site Admin
Messaggi: 970
Iscritto il: 14 mag 2006, 17:51
Località: Paris
Contatta:

Messaggio da Nonno Bassotto »

Se volete quella che conosco io è diversa, e mostra in effetti che per ogni campo K e per ogni gruppo moltiplicativo finito G<K*, G è ciclico.
The best argument against democracy is a five-minute conversation with the average voter. - Winston Churchill
sqrt2
Messaggi: 142
Iscritto il: 19 gen 2006, 14:43
Località: Genova

Messaggio da sqrt2 »

@ moebius(l'esperto nanologo :wink: ): ho postato quest'esercizio perchè mi piaceva, e mi piace condividere un bell'esercizio, senza pretese di originalità (tra l'altro non rientra nel programma dei corsi di algebra che ho seguito).
Poi, se il problema non è la difficoltà ma il fatto che sia ben noto l'esercizio, mi dispiace.
Grazie per avermelo detto comunque, almeno capisco il motivo delle nonrisposte.

@ nonno bassotto: la dimostrazione generale che conosci tu si fonda sempre su considerazioni sull'elemento di ordine massimo?
Avatar utente
Nonno Bassotto
Site Admin
Messaggi: 970
Iscritto il: 14 mag 2006, 17:51
Località: Paris
Contatta:

Messaggio da Nonno Bassotto »

No, si basa sulla classificazione dei gruppi abeliani finiti.
The best argument against democracy is a five-minute conversation with the average voter. - Winston Churchill
Avatar utente
hydro
Messaggi: 219
Iscritto il: 07 apr 2005, 17:11
Località: milano

Messaggio da hydro »

Nonno Bassotto ha scritto:Se volete quella che conosco io è diversa, e mostra in effetti che per ogni campo K e per ogni gruppo moltiplicativo finito G<K*, G è ciclico.
Io conosco una dimostrazione di questo fatto (nel caso G=K*) che fa uso del seguente lemma (vi invito a dimostrarlo):

Se G è un gruppo abeliano in cui ogni elemento ha periodo finito, ed esiste il massimo m dei periodi degli elementi di G, allora g^m=1 per ogni g in G.
darkcrystal
Messaggi: 706
Iscritto il: 14 set 2005, 11:39
Località: Chiavari

Messaggio da darkcrystal »

Vediamo, non ho mai fatto dimostrazioni coi gruppi, ma bisogna pur iniziare, e questa mi sembra abbastanza semplice...
Diciamo che esista un elemento di ordine p, $ p \not | m $. Sappiamo poi che esiste un elemento di ordine m. Ma allora è noto che esiste un elemento di ordine $ lcm(p,m) > m $, assurdo perchè m dovrebbe essere il massimo degli ordini.
"Solo due cose sono infinite: l'universo e la stupidità dell'uomo, e non sono tanto sicuro della prima" - Einstein

Membro dell'EATO
sqrt2
Messaggi: 142
Iscritto il: 19 gen 2006, 14:43
Località: Genova

Messaggio da sqrt2 »

Mi è capitato per caso di trovare nelle dispense olimpiche di Gobbino un teorema che, riformulato in termini di gruppi, asserisce che
$ \mathbb{Z}_n^* $ rispetto al prodotto è un gruppo ciclico se e solo se $ n=2,4,p^n,2p^n $ con $ p $ primo dispari.
Qualcuno saprebbe dimostrarlo o conosce una dimostrazione?
Avatar utente
Marco
Site Admin
Messaggi: 1331
Iscritto il: 01 gen 1970, 01:00
Località: IMO '93

Messaggio da Marco »

Beh, di nuovo, e' un fatto che, se hai l'armamentario di Algebra I, e' ben noto. Richiede, ad esempio un po' di teoria dei campi finiti, che usi per dire che $ \mathbf Z_q^* $ e' ciclico, quando $ q $ e' potenza di primo dispari.

Questa e' l'unica parte non elementare, ed a quel punto e' [quasi] fatta, perche' il resto viene dal Teorema Cinese del Resto. Dico quasi, perche' resta fuori il primo 2, che e' rognosetto.

Ma si vede a mano che $ \mathbf Z_8^* $ e' non ciclico, e che per le potenze maggiori di 8, i gruppi successivi devono avere questo gruppo non ciclico come quoziente.
[i:2epswnx1]già ambasciatore ufficiale di RM in Londra[/i:2epswnx1]
- - - - -
"Well, master, we're in a fix and no mistake."
Avatar utente
edriv
Messaggi: 1638
Iscritto il: 16 feb 2006, 19:47
Località: Gradisca d'Isonzo
Contatta:

Messaggio da edriv »

darkcrystal ha scritto:Vediamo, non ho mai fatto dimostrazioni coi gruppi, ma bisogna pur iniziare, e questa mi sembra abbastanza semplice...
Diciamo che esista un elemento di ordine p, $ p \not | m $. Sappiamo poi che esiste un elemento di ordine m. Ma allora è noto che esiste un elemento di ordine $ lcm(p,m) > m $, assurdo perchè m dovrebbe essere il massimo degli ordini.
Sembra che tutti trovino chiara questa dimostrazione, io no però... come hai definito m? p è lo stesso p di $ ~ \mathbb{Z}_p $? Perchè mi sembra che hai dimostrato che l'ordine di un elemento divide l'ordine massimo degli elementi?
sqrt2
Messaggi: 142
Iscritto il: 19 gen 2006, 14:43
Località: Genova

Messaggio da sqrt2 »

@ edriv: darkcrystal rispondeva a hydro, il cui post magari non hai visto
hydro ha scritto:Se G è un gruppo abeliano in cui ogni elemento ha periodo finito, ed esiste il massimo m dei periodi degli elementi di G, allora g^m=1 per ogni g in G.
Quindi m è definito sopra, p non è il p di $ \mathbb{Z}_p $ e darkcrystal dimostra proprio quello che dici.
sqrt2
Messaggi: 142
Iscritto il: 19 gen 2006, 14:43
Località: Genova

Messaggio da sqrt2 »

Riprendo questo vecchio post, che mi è ricapitato tra le mani. Grazie a Marco e Gian per i suggerimenti e gli aiuti.
Vi chiedo di correggere eventuali sbavature e di completare la dimostrazione del secondo punto.
Buona lettura!
_________________________________________________________________________

$ U_n := (U(\mathbb{Z}_n),*) $ è ciclico se e solo se $ n = 2, 4, p^m, 2p^m $ con $ p $ primo dispari.

-$ n = p $ con p primo --> $ U_p $ ciclico.

Infatti, poichè $ U_p $ è un gruppo abeliano finito con p-1 elementi, è isomorfo a $ \mathbb{Z}_{q_1^{a_1}}*...*\mathbb{Z}_{q_s^{a_s}} $, con $ q_i^{a_i} $ invarianti primari (cioè primi non necessariamente distinti tra loro) tali che il loro prodotto sia p-1.
Ma in questo caso si ha che i $ q_i^{a_i} $ sono tutti distinti, e quindi si ha la tesi, poichè in tal caso $ U_p $ è isomorfo a $ \mathbb{Z}_{p-1} $, che è ciclico.
Infatti, se per assurdo ci fossero due invarianti primari non distinti, che, senza perdita di generalità, possiamo supporre essere $ q_1 $ e $ q_2 $, allora $ (h q_1^{a_1-1},k q_2^{a_2-1},0,..,0) $, con $ h, k = 0,...,p-1 $ sarebbero $ p^2 $ soluzioni dell'equazione $ x^p = 1 $ (che in notazione additiva diventa $ px = 0 $), ma $ \mathbb{Z}_p $ è un campo, quindi $ x^p = 1 $ ha alpiù n radici per Ruffini, assurdo.

-$ n = p^m $ con p primo --> $ U_n $ ciclico.

Se m = 2, per il primo teorema di omomorfismo tra gruppi e il lemma si ha che $ U_{p^2} / Ker( \pi) = U_p = \mathbb{Z}_{p-1} $, ma allora $ U_{p^2} = \mathbb{Z}_{p-1} * \mathbb{Z}_{p} = \mathbb{Z}_{p^2-p} $.
Ora se m > 2, procedo per induzione: per il lemma e il primo teorema di omomorfismo si ha che o $ U_{p^{m+1}} = U_{p^m} * \mathbb{Z}_p = \mathbb{Z}_{p^m-p^{m-1}} * \mathbb{Z}_{p} = \mathbb{Z}_{p} * \mathbb{Z}_{p^{m-1}} * \mathbb{Z}_{p-1} $ o
$ U_{p^{m+1}} = \mathbb{Z}_{p^{m}} * \mathbb{Z}_{p-1} = \mathbb{Z}_{p^m-p^{m-1}} $.
Si tratta di escludere il primo caso (COMPLETARE!)

-$ n = 2 p^m $ con p primo --> $ U_n $ ciclico.

Infatti, per il lemma, la proiezione canonica da $ U_{2p^m} $ a $ U_{p^m} $ è un omomorfismo surgettivo di gruppi con lo stesso numero di elementi, poichè $ \phi(2 p^m) = \phi(p^m) = p^m - p^{m-1} $, perciò è un isomorfismo, e dunque anche il primo insieme è ciclico.

-$ n = 4 $ --> $ U_4 $ è ciclico (3 lo genera).

-$ n = 2^m * k $ con m>2 --> $ U_n $ non ciclico.

Infatti si verifica a mano che per n=8 (tutti gli elementi han ordine al più 2), quindi se per assurdo per qualche m il gruppo $ U_{2^m} $ fosse ciclico, sfruttando il lemma avrei, per il primo teorema di omomorfismo di anelli, che $ U_{2^m*k} / Ker( \pi) = U_{8} $, ma un gruppo ciclico e uno non ciclico non possono essere isomorfi (e il quoziente di un gruppo ciclico è ciclico).

-$ n = 4^b * p_1^{a_1} *...* p_s^{a_s} $ con $ s>=2 $ e $ b \in \{0,1\} $ o $ s=1 $ e $ b=1 $, $ p_i $ primi dispari --> $ U_n $ non ciclico.

$ \phi(4) $ e $ \phi(p_i^{a_i}) $ sono pari, perciò preso un elemento di $ U_n $, $ x^{\frac{\phi(n)}{2}} = x^{\frac{\phi(4^b) \phi(p_1^{a_1}) ... \phi(p_s^{a_s})}{2}} = 1 $ $ mod (p_i^{a_i}) $ e $ mod (4) $ (se b=1) per il piccolo teorema di fermat, dunque l'ordine di ogni elemento di $ U_n $ è al più $ \phi(n)/2 $, dunque non c'è nessun generatore.

Lemma: $ \pi:U_n\mapsto U_d $, proiezione canonica, è un omomorfismo surgettivo di gruppi con $ d|n $.

Infatti la proiezione canonica da $ \mathbb{Z}_n $ in $ \mathbb{Z}_d $ è un omomorfismo di anelli surgettivo, e un elemento è invertibile rispetto al prodotto in $ \mathbb{Z}_n $ (se e solo se è coprimo con n, se e solo se è coprimo con d per ogni $ d|n $,) se e solo se è invertibile in $ \mathbb{Z}_d $ per ogni $ d|n $.
"Dio ha creato i numeri interi, tutto il resto è opera dell'uomo"
"I matematici parlano con Dio, i fisici parlano ai matematici, gli altri parlano tra loro"
Rispondi